2002 AMC 12A Problems/Problem 2

Revision as of 14:35, 17 October 2007 by 1=2 (talk | contribs)

Problem

Cindy was asked by her teacher to subtract 3 from a certain number and then divide the result by 9. Instead, she subtracted 9 and then divided the result by 3, giving an answer of 43. What would her answer have been had she worked the problem correctly?

$\mathrm{(A) \ } 15\qquad \mathrm{(B) \ } 34\qquad \mathrm{(C) \ } 43\qquad \mathrm{(D) \ } 51\qquad \mathrm{(E) \ } 138$


Solution

$\dfrac{x-9}{3}=43$

$x=43*3+9=138$

$\dfrac{x-3}{9}=15 \Rightarrow \mathrm {(A)}$

See Also

2002 AMC 12A (ProblemsAnswer KeyResources)
Preceded by
Problem 1
Followed by
Problem 3
1 2 3 4 5 6 7 8 9 10 11 12 13 14 15 16 17 18 19 20 21 22 23 24 25
All AMC 12 Problems and Solutions